LSAT and Law School Admissions Forum

Get expert LSAT preparation and law school admissions advice from PowerScore Test Preparation.

 Jon Denning
PowerScore Staff
  • PowerScore Staff
  • Posts: 904
  • Joined: Apr 11, 2011
|
#19945
It depends on what you mean by "correct." That is, how you classify those various statements as either "known premise," "conclusion," and "given answer choice" will dictate what you can do with them!

In the first, if A :arrow: B is a premise, and A :arrow: C is the conclusion we're trying to prove, then we need an answer that forms the chain A :arrow: B :arrow: C, meaning we need the B :arrow: C statement.

In the second, if A :arrow: B is a premise and A :arrow: C the conclusion, then adding C :arrow: B wouldn't tell us anything.

BUT, if we had A :arrow: C as premise and A :arrow: B as conclusion then the chain you show with A :arrow: C :arrow: B (made by adding C :arrow: B) would prove the A :arrow: B conclusion.

So it all depends on how you're using/given each piece and what you're trying to do.
 ShannonOh22
  • Posts: 70
  • Joined: Aug 15, 2019
|
#67942
Hi Powerscore,

I fully see how E is correct on this question, but I'm struggling to figure out why D is incorrect.

Here is my pattern of reasoning:

inspected -> infected -> safe to eat

According to the contrapositive, wouldn't this also equal

safe to eat-> infected -> inspected

This is what answer choice D is saying, and I thought whenever we have a contrapositive, it is typically that answer choice that is correct b/c the LSAT writers like to make things as difficult as possible.

Have I gone wrong somewhere in my logic deduction? Please help!! Thanks! :)
 Jeremy Press
PowerScore Staff
  • PowerScore Staff
  • Posts: 1000
  • Joined: Jun 12, 2017
|
#67968
Hi Shannon,

The first chain you drew is accurate, and so is your drawing of the contrapositive of that chain. The problem with answer choice D is that it actually reverses the first two elements of your (correctly drawn) contrapositive chain. Answer choice D states, "It is not safe to eat any fruit that is infected." "Any" is a sufficient condition indicator, thus answer choice D diagrams as: infected :arrow: NOT safe to eat. This is the Mistaken Reversal of what you'd need to fill out your contrapositive chain, thus it cannot justify the reasoning in the stimulus. Make sense?

I hope this helps!

Jeremy
 lsatbossintraining
  • Posts: 27
  • Joined: Oct 21, 2019
|
#71758
Here’s my diagram using the double-not: could someone tell me where I’m going wrong?

Safe to Eat <—Inspected<—|—>Infected—>Rotten

s
Inferences: Safe to Eat<—>~Infected, ~Rotten

None of the answer choices seem to fit my inference or general diagram.

Thanks much.
 Adam Tyson
PowerScore Staff
  • PowerScore Staff
  • Posts: 5153
  • Joined: Apr 14, 2011
|
#71839
You have diagrammed all the relationships correctly, boss, but you have to separate what is a premise (If inspected then not infected, and if infected then rotten) from what is a conclusion (if inspected then safe). We are looking for a new premise that will PROVE that if inspected, then safe, when all we know about inspected fruit is that it is not infected. The missing link is "and if it is not infected then it is safe." If we add that premise to the argument the conclusion becomes inescapable, proven...justified. That's what answer E does.

Your prephrase contains a double-arrow relationship (Safe :dblline: Infected) that would do the job, although it's more than we need. We only need to know that any fruit that is not infected is safe - we do NOT need to know that any fruit that is safe is not infected. There could be some safe but infected fruit, as long as all the uninfected fruit is definitely safe. So, you had a perfectly good justify answer built in to your prephrase, but by looking for something stronger than was required you may have inadvertently rejected a perfectly acceptable answer choice.

Get the most out of your LSAT Prep Plus subscription.

Analyze and track your performance with our Testing and Analytics Package.